ChaseDream
搜索
12
返回列表 发新帖
楼主: 八戒
打印 上一主题 下一主题

LSAT-7-4-23/LSAT-7-4-24

[复制链接]
11#
发表于 2004-4-20 18:57:00 | 只看该作者
ding有人回答嘛?
12#
发表于 2004-4-21 10:39:00 | 只看该作者

题干前提:no increase wage——》strike


               increase wage——》sell subsidirary


结论:sell subsidiary


A取非,will go on strike


E为什么错?confusing


其实你自己已经把逻辑关系分析得很清楚了,为使结论成立,必须increase wage,那么如何increase wage呢,只有加入 not strike这个条件。所以这是个assumption


13#
发表于 2007-5-11 17:28:00 | 只看该作者
以下是引用dorbear在2003-12-7 6:49:00的发言:
[face=Verdana]In Order To 有两种情形;一种表达必要条件的逻辑关系,通常是In Order to, Must/Have to; 另外一种不表达逻辑关系,只是表达目的和手段(XDF的逻辑教学在这个地方是错误的)。因此后者并不存在一种充分条件或必要条件的成分在其中。以OG为例子:

132. Passengers must exit airplanes swiftly after accidents, since gases released following accidents are toxic to humans and often explode soon after being released. In order to prevent passenger deaths from gas inhalation, safety officials recommend that passengers be provided with smoke hoods that prevent inhalation of the gases.

Which of the following, if true, constitutes the strongest reason not to require implementation of the safety officials' recommendation?
(A) Test evacuations showed that putting on the smoke hoods added considerably to the overall time it took passengers to leave the cabin.
(B) Some airlines are unwilling to buy the smoke hoods because they consider them to be prohibitively expensive.
(C) Although the smoke hoods protect passengers from the toxic gases, they can do nothing to prevent the gases from igniting.
(D) Some experienced flyers fail to pay attention to the safety instructions given on every commercial flight before takeoff.
(E) In many airplane accidents, passengers who were able to reach emergency exits were overcome by toxic gases before they could exit the ariplane.

这是一个典型的目的和措施类型,目的和措施之间既没有必要,也没有充分的逻辑关系。而进行削弱或加强,只要就措施的有效性举出一个正面或反面的例子就可以了。

跟上面相反,LSAT的例子确实前一种情况,有逻辑关系的表达In Order To.... Have to. 了解了以上的区别,就不难判断出各自正确的选项。

23. The workers at Bell Manufacturing will shortly go on strike unless the management increases their wages. As Bell's president is well aware, however, in order to increase the worker's wages, Bell would have to sell off some of its subsidiaries. So, some of Bell's subsidiaries will be sold.
The conclusion above is properly drawn if which one of the following is assumed?
(A) Bell Manufacturing will begin to suffer increased losses.
(B) Bell's management will refuse to increase its worker's wages.
(C) The workers at Bell Manufacturing will not be going on strike.
(D) Bell's president has the authority to offer the workers their desired wage increase.
(E) Bell's workers will not accept a package of improved benefits in place of their desired wage increase. [/face]

好厉害哦~
14#
发表于 2007-5-12 04:10:00 | 只看该作者

The workers at Bell Manufacturing will shortly go on strike unless the management increases their wages. As Bell's president is well aware, however, in order to increase the worker's wages, Bell would have to sell off some of its subsidiaries. So, some of Bell's subsidiaries will be sold.

 

 

This is a formal logic question. First you have to translate stimulus to simble.

 

 

Not strike -à Increase wage
Increase wage -
à sell off some of its subsidiaries

--------------------------------------------------------------------------

Conclusion: some of Bell's subsidiaries will be sold.

 

 

From above you can see that there is a big Gap between premises and conclusion. The Premises we have, do not lead us to the conclusion. We need to add premises/assumption to make the argument valid.

 

 

The conclusion above is properly drawn if which one of the following is assumed?
(A) Bell Manufacturing will begin to suffer increased losses.
(B) Bell's management will refuse to increase its worker's wages.
(C) The workers at Bell Manufacturing will not be going on strike.

If we add answer C: Not strike to the premise. It will lead to conclusion. So, it the assumption of this argument.

 

 

Premises: Not strike à Increase wages à sell off some of its subsidiaries

Not strike -à Increase wage
Increase wage -
à sell off some of its subsidiaries

--------------------------------------------------------------------------

Conclusion: some of Bell's subsidiaries will be sold.

 
(D) Bell's president has the authority to offer the workers their desired wage increase.
(E) Bell's workers will not accept a package of improved benefits in place of their desired wage increase.


[此贴子已经被作者于2007-5-12 4:12:08编辑过]
您需要登录后才可以回帖 登录 | 立即注册

Mark一下! 看一下! 顶楼主! 感谢分享! 快速回复:

所属分类: 法学院申请

近期活动

正在浏览此版块的会员 ()

手机版|ChaseDream|GMT+8, 2024-4-19 20:13
京公网安备11010202008513号 京ICP证101109号 京ICP备12012021号

ChaseDream 论坛

© 2003-2023 ChaseDream.com. All Rights Reserved.

返回顶部